Do All Bounded Monotone Sequences Converge?

  • Thread starter Thread starter kmikias
  • Start date Start date
  • Tags Tags
    Bounded Sequence
Click For Summary
A bounded monotone sequence, whether increasing or decreasing, is proven to converge using the least upper bound (LUB) and greatest lower bound (GLB) properties. For an increasing sequence, it is shown that as n approaches infinity, the terms approach the limit L within an epsilon range. The discussion highlights confusion in proving convergence for a bounded monotone decreasing sequence, particularly in establishing the relationship between the terms and the limit L. The participant struggles with the implications of the GLB, specifically regarding whether the terms can be bounded above by L + ε. Overall, the convergence of bounded monotone sequences is affirmed, but clarity is needed in the decreasing case.
kmikias
Messages
72
Reaction score
0

Homework Statement


A bounded monotone sequence converges.
Proof
for bounded monotone increasing sequence and decreasing sequence.
Does both them converges?

Homework Equations


So, I used the least upper bound and great lower bound to prove increasing sequence and decreasing sequence.

Property of LUB and GREAT LOWER BOUND.

The Attempt at a Solution



a bounded monotone increasing sequence to converge...
Proof.
a_{n} is monotone increaing if n > N(ε), then a_{n}≥ a_{N(ε)} > L -ε. But a_{n) ≤ L.
thus L - ε < a_{n} ≤ L for n > N(ε); that is | a_{n} - L | < ε for n>N(ε). Δ

Proof for a bounded monotone decreasing sequence to converge..
this is where i got lost.

so i used great lower bound to do the proof.
we know G.L.B has this two property
1. a_{n} ≥ L for every n
2. for ε > 0, there exist a positive number N(ε) SUCH THAT a_{N(ε)} < L-ε

so
a_{n} is monotone decresing if n > N(ε), then L ≤ a_{n} ≤ L +ε. am kind of lost here.
 
Last edited:
Physics news on Phys.org
Additional to the question i asked,

a_{N(E)} < L - E
THEN
a_{(N(E)} < L - E < a_{n}
but we know
a_{n} >= L
THEN
L <= a_{n} <= L+E

BUT I STILL HAVE PROBLEM BECAUSE I DON'T KNOW IF a_{n} <= L+E IS TRUE.
 
Question: A clock's minute hand has length 4 and its hour hand has length 3. What is the distance between the tips at the moment when it is increasing most rapidly?(Putnam Exam Question) Answer: Making assumption that both the hands moves at constant angular velocities, the answer is ## \sqrt{7} .## But don't you think this assumption is somewhat doubtful and wrong?

Similar threads

  • · Replies 8 ·
Replies
8
Views
2K
  • · Replies 7 ·
Replies
7
Views
2K
  • · Replies 9 ·
Replies
9
Views
2K
Replies
6
Views
3K
  • · Replies 3 ·
Replies
3
Views
1K
Replies
7
Views
4K
  • · Replies 14 ·
Replies
14
Views
5K
Replies
3
Views
2K
  • · Replies 3 ·
Replies
3
Views
2K
  • · Replies 13 ·
Replies
13
Views
2K